Does the gradiant change when stretching coordinates?

  • Thread starter Thread starter physicsfreak88
  • Start date Start date
  • Tags Tags
    Change Coordinates
Click For Summary
When stretching coordinates in a Cartesian system by a factor of 2, the gradient remains unchanged as it represents the rate of change of a function. This can be verified by substituting the stretched coordinates into a known problem and observing the differentiation process. The discussion also touches on the analogy of converting speed units, questioning if a car traveling at x m/s is equivalent to traveling at x mph, highlighting the importance of understanding unit conversions. Overall, the gradient's consistency under coordinate transformation emphasizes its fundamental nature in calculus. The relationship between gradients and coordinate transformations is crucial for accurate mathematical modeling.
physicsfreak88
Messages
8
Reaction score
0

Homework Statement



Well, if you have given a gradiant in cartisian coordinate system? what happened to the gradiant if we stretched the coordinates by factor of 2?

Homework Equations

The Attempt at a Solution


I think gradiant should be the same as it's the rate of change of some function.
 
Physics news on Phys.org
You could test this yourself by taking a known problem where you have the gradient and subbing in 2x for x and see how it flows through the differentiations to get the gradient.
 
physicsfreak88 said:

Homework Statement



Well, if you have given a gradiant in cartisian coordinate system? what happened to the gradiant if we stretched the coordinates by factor of 2?

Homework Equations

The Attempt at a Solution


I think gradiant should be the same as it's the rate of change of some function.

If a car is traveling at x m/s, is it traveling at x mph?
 
Question: A clock's minute hand has length 4 and its hour hand has length 3. What is the distance between the tips at the moment when it is increasing most rapidly?(Putnam Exam Question) Answer: Making assumption that both the hands moves at constant angular velocities, the answer is ## \sqrt{7} .## But don't you think this assumption is somewhat doubtful and wrong?

Similar threads

Replies
1
Views
1K
  • · Replies 1 ·
Replies
1
Views
2K
  • · Replies 14 ·
Replies
14
Views
2K
  • · Replies 8 ·
Replies
8
Views
2K
  • · Replies 7 ·
Replies
7
Views
2K
  • · Replies 3 ·
Replies
3
Views
2K
  • · Replies 3 ·
Replies
3
Views
2K
  • · Replies 6 ·
Replies
6
Views
3K
  • · Replies 4 ·
Replies
4
Views
2K
Replies
4
Views
2K